Матричные элементы оператора импульса в позиционном представлении

У меня есть два связанных вопроса о представлении оператора импульса в позиционном базисе.

Действие оператора импульса на волновую функцию заключается в ее выводе:

п ^ ψ ( Икс ) "=" я ψ ( Икс ) Икс

(1) Можно ли сделать из этого вывод, что:

Икс | п ^ | Икс "=" я дельта ( Икс Икс ) Икс ?

И что означает это выражение?

(2) Используя уравнения:

Икс | Икс ^ п ^ | Икс Икс "=" Икс | п ^ Икс ^ | Икс Икс "=" Икс | п ^ | Икс

и

Икс | [ Икс ^ , п ^ ] | Икс "=" я дельта ( Икс Икс )

можно сделать вывод, что

Икс | п ^ | Икс "=" я дельта ( Икс Икс ) Икс Икс

Это уравнение подходит? следует ли из этого

дельта ( Икс Икс ) Икс "=" дельта ( Икс Икс ) Икс Икс ?

Если вам нравится этот вопрос, вы также можете прочитать этот пост.
Также будьте осторожны при делении на Икс Икс поскольку это определяется только тогда, когда Икс Икс . На самом деле, обратите внимание, что @Qmechanic тщательно записывает свои тождества, никогда не разделяя их на Икс Икс .
Спасибо за ответы на все вопросы. См. Также ответ Рона Маймона здесь (в вопросе, предложенном @Qmechanic), который также в значительной степени отвечает на мой вопрос.

Ответы (3)

1) Обратите внимание, что, вставив полный набор состояний позиции, мы можем написать

п ^ ψ ( Икс ) "=" Икс | п ^ | ψ "=" г Икс Икс | п ^ | Икс Икс | ψ "=" г Икс Икс | п ^ | Икс ψ ( Икс )
поэтому, если мы установим
Икс | п ^ | Икс "=" я Икс дельта ( Икс Икс ) "=" я Икс дельта ( Икс Икс )
то мы можем использовать интегрирование по частям, чтобы получить
п ^ ψ ( Икс ) "=" я г Икс Икс дельта ( Икс Икс ) ψ ( Икс ) "=" я г Икс дельта ( Икс Икс ) г ψ г Икс ( Икс ) "=" я г ψ г Икс ( Икс )
Так что ваше выражение верно. Производная дельта-функции по существу определяется интегрированием по частям, которое я только что выполнил; на самом деле производные распределений вообще определяются аналогичным образом. Посмотрите, например, эту лекцию .

Надеюсь, это поможет; дайте мне знать о любых опечатках!

Ваше здоровье!

1) Пользователь joshphysics уже правильно ответил на 1-й вопрос ОП.

2a) Что касается второго вопроса ОП, то получается

я дельта ( Икс Икс )   "="   я Икс | Икс   "="   Икс | [ Икс ^ , п ^ ] | Икс   "="   Икс | Икс ^ п ^ | Икс Икс | п ^ Икс ^ | Икс
(А)   "="   ( Икс Икс ) Икс | п ^ | Икс   "=" ( 1 )   я ( Икс Икс ) Икс дельта ( Икс Икс ) .

Другими словами,

(Б) дельта ( Икс Икс )   "="   ( Икс Икс ) Икс дельта ( Икс Икс ) ,

что также следует путем дифференцирования тождества

(С) ( Икс Икс ) дельта ( Икс Икс )   "="   0

относительно Икс .

2б) уравнение (B) не должны быть разделены на обе стороны относительно. Икс Икс . Проблема в том, что распределение 1 Икс дельта ( Икс ) плохо определен.

Один из аргументов, почему это так, выглядит примерно следующим образом. Напомним, что один из способов понять распределение ты заключается в оценке на гладких тестовых функциях г : р С . Например, если распределение ты есть дельта-распределение Дирака , то по определению

(Д) ты [ г ]   "="   г ( 0 ) ,

или, что то же самое, в, возможно, более знакомой нотации,

(Е) р г Икс   дельта ( Икс ) г ( Икс )   "="   г ( 0 ) .

Можно вообще не умножать 1 два распределения, но можно умножить гладкую функцию ф : р С с раздачей ты . Продукт ф ты по определению

(Ф) ( ф ты ) [ г ]   "="   ты [ ф г ] .

Так что если ты есть дельта-распределение Дирака, получается

(Г) ( ф ты ) [ г ]   "="   ф ( 0 ) г ( 0 ) .

В случае OP, если мы попытаемся установить ф ( Икс ) "=" 1 Икс , затем ф ( 0 ) было бы нечетко определено.

Другой менее формальный аргумент заключается в том, что если мы ошибочно примем 1 Икс дельта ( Икс ) как распределение, то мы склонны к, казалось бы, бессмысленным противоречиям а-ля

Икс ( 1 Икс дельта ( Икс ) )   "="   Икс ( 1 Икс дельта ( Икс ) )   "="   ( Икс 1 Икс ) дельта ( Икс )
(ЧАС)   "="   ( 1 Икс Икс ) дельта ( Икс )   "="   1 Икс ( Икс дельта ( Икс ) )   "="   1 Икс 0   "="   0 , (Неправильный!)

т.е. мы потеряли ассоциативность умножения.

--

1 Мы игнорируем теорию Коломбо . См. также этот пост mathoverflow.

Эта дополнительная деталь была очень хороша; Я нахожу все ваши сообщения о дистрибутивах полезными; спасибо Qмеханик.

@joshphysics дал отличную иллюстрацию того, почему ваша первая часть, то есть ⟨x|p^|x′⟩=−iℏ∂δ(x−x′)∂x? согласуется с квантовой механикой;

Давайте проверим вашу вторую часть достаточно интуитивно.

Так как в целом:

Икс г ( Икс ) ф ( Икс ) г Икс "=" ф ( Икс ) г г Икс ( Икс г ( Икс ) ) г Икс "=" ф ( Икс ) ( Икс г ( Икс ) + г ( Икс ) ) г Икс
Если
ф ( Икс ) "=" дельта ( Икс )
Мы заключаем, что:

ф ( Икс ) ( Икс г ( Икс ) + г ( Икс ) ) г Икс "=" дельта ( Икс ) г ( Икс ) г Икс "=" Икс г ( Икс ) дельта ( Икс ) г Икс

Таким образом

дельта ( Икс ) "=" 1 Икс дельта ( Икс )

Правда в математике